[Physics] Proving that the Boltzmann entropy is equal to the thermodynamic entropy

entropystatistical mechanicstemperaturethermodynamics

I've been trying to understand how we can equate the Boltzmann entropy $k_B \ln \Omega$ and the entropy from thermodynamics. I'm following the approach found in the first chapter in Pathria's statistical mechanics, and in many other texts. Many other questions on stackexchange come close to addressing this problem, but I don't think any of the answers get at my specific question.

So, we're considering two isolated systems 1 and 2, which are brought into thermal contact and allowed to exchange energy (let's assume for simplicity that they can only exchange energy). On the thermodynamic side of the problem, we have the necessary and sufficient condition for thermal equilibrium

$$T_1=\frac{\partial E_1}{\partial S_1}=T_2=\frac{\partial E_2}{\partial S_2},$$

where the temperatures $T_1$ and $T_2$, the internal energies $E_1$ and $E_2$, and the entropies $S_1$ and $S_2$ are all defined appropriately in operational, thermodynamic terms. On the other hand, we can show that the necessary and sufficient condition for equilibrium from the standpoint of statistical mechanics is given by

$$\beta_1 \equiv \frac{\partial \ln \Omega_1}{\partial E_1}= \beta_2 \equiv \frac{\partial \ln \Omega_2}{\partial E_2}.$$

Here, $\Omega_1$ and $\Omega_2$ are the number of microstates associated with the macrostate of each system. Now, since both of these relations are necessary and sufficient for equilibrium, one equality holds if and only if the other also holds. My question is: How can we proceed from here to show that $S=k_B \ln \Omega$, without limiting our scope to specific examples (like an ideal gas)? In Pathria's text and in other treatments, I don't see much explanation for how this step is justified.

My possibly wrong thoughts are: It seems like we first need to show that $\beta$ is a function of $T$ alone (and indeed the same function of $T$ for both systems), and then show that the form of this function is in fact $\beta \propto T^{-1}$. But I'm not sure how to prove either of those claims.

Best Answer

$\newcommand{\mean}[1] {\left< #1 \right>}$ $\DeclareMathOperator{\D}{d\!}$ $\DeclareMathOperator{\pr}{p}$

Proof that $\beta = \frac{1}{k T}$ and that $S = k \ln \Omega$

This proof follows from only classical thermodynamics and the microcanonical ensemble. It makes no assumptions about the analytic form of statistical entropy, and does not involve the ideal gas law.

First recall that the pressure of an individual microstate is given from mechanics as:

\begin{align} P_i &= -\frac{\D E_i}{\D V} \end{align}

When assuming only $P$-$V$ mechanical work, the energy of a microstate $E_i(N,V)$ is only dependent on two variables, $N$ and $V$. For example, consider a quantum mechanical system like particles confined in a box. Therefore, at constant composition $N$,

\begin{align} P_i &= -\left( \frac{\partial E_i}{\partial V} \right)_N \end{align}

In a system described by the microcanonical ensemble, there are $\Omega$ possible microstates of the system. The energy of an individual microstate $E_i$ is likewise trivially independent of the number microstates $\Omega$ in the ensemble. Therefore, the pressure of an individual microstate can also be expressed as

\begin{align} P_i &= -\left( \frac{\partial E_i}{\partial V} \right)_{\Omega,N} \end{align}

According to statistical mechanics, the macroscopic pressure of a system is given by the statistical average of the pressures of the individual microstates:

\begin{align} P = \mean{P} &= \sum_i^\Omega \pr_i P_i \end{align}

where $\pr_i$ is the equilibrium probability of microstate $i$. For a microcanonical ensemble, all microstates have the same energy $E_i = E$, where $E$ is the energy of the system. Therefore, from the fundamental assumption of statistical mechanics, all microcanonical microstates have the same probability at equilibrium

\begin{align} \pr_i = \frac{1}{\Omega} \end{align}

It follows that the pressure of a microcanonical system is given by

\begin{align} P = \mean{P} &= -\sum_i^\Omega \frac{1}{\Omega} \left( \frac{\partial E_i}{\partial V} \right)_{\Omega,N} \\ &= -\frac{1}{\Omega} \sum_i^\Omega \left( \frac{\partial E}{\partial V} \right)_{\Omega,N} \\ &= -\frac{\Omega}{\Omega} \left( \frac{\partial E}{\partial V} \right)_{\Omega,N} \\ P &= -\left( \frac{\partial E}{\partial V} \right)_{\Omega,N} \end{align}

This expression for the pressure of a microcanonical system can be compared to the classical expression

\begin{align} P &= -\left( \frac{\partial E}{\partial V} \right)_{S,N} \end{align}

which immediately suggests a functional relationship between entropy $S$ and $\Omega$.

Now we take the total differential of the energy of a microcanonical system:

\begin{align} \D E = \left(\frac{\partial E}{\partial \ln \Omega}\right)_{V, N} \D \ln\Omega + \left(\frac{\partial E}{\partial V}\right)_{\ln \Omega, N} \D V \end{align}

As stated in the OP, for the microcanonical ensemble, the condition for thermal equilibrium is:

\begin{align} \beta &= \left( \frac{\partial \ln \Omega}{\partial E} \right)_{V,N} \end{align}

Thus,

\begin{align} \D E = \frac{1}{\beta} \D \ln\Omega - P \D V \end{align}

Compare with the classical first law of thermodynamics:

\begin{align} \D E = T \D S - P \D V \end{align}

Because these equations are equal, we see that

\begin{align} T \D S &= \frac{1}{\beta} \D \ln\Omega \\ \D S &= \frac{1}{T \beta} \D \ln\Omega \end{align}

Note that both $\D S$ and $\D \ln\Omega$ are exact differentials, so $\frac{1}{T \beta}$ must be either a constant or a function of $\Omega$. Since $\D S$ and $\D \ln\Omega$ are both extensive quantities, $\frac{1}{T \beta}$ cannot depend on $\Omega$, and therefore

\begin{align} k &= \frac{1}{T \beta} \\ \beta &= \frac{1}{k T} \end{align}

where $k$ is a universal constant that is independent of composition, since $\beta$ and $T$ are both independent of composition.

By integrating, we have

\begin{align} S &= k \ln\Omega + C \end{align}

where $C$ is a constant that is independent of $E$ and $V$, but may depend on $N$. By invoking the third law we can set $C=0$ to arrive at the famous Boltzmann expression for the entropy of a microcanonical system:

\begin{align} S &= k \ln\Omega \end{align}

Related Question